This page shows a recording of a live class. We're working hard to create our standard, concise explanation videos for the questions in this PrepTest. Thank you for your patience!

Here we have a weakening question, as the stem asks us: The charge made above against alternative medicine is most seriously weakened if it is true that

The question begins with the claim that orthodox medicine is ineffective at both ends of the spectrum of ailments. Orthodox in this context just means ‘standard’. This claim is elaborated with the details that orthodox medicine is bad at dealing with both minor discomfort and serious life-threatening illnesses. So standard medical practice can’t do much for your toothache or your tumor, if your problem is mild they can’t treat symptoms and if it’s severe they can’t cure it. For this reason, people turn to alternative medicine because orthodox medicine fails them or produces unwanted side effects. But alternative medicine has less unwanted side effects partly because it doesn’t have any effects at all. Doesn’t sound like it’s going to do any better of a job! Our task is to weaken this conclusion that alternative medicine doesn’t have any positive or negative effects. Let’s see our answer choices:

Answer Choice (A) This doesn’t have anything to do with effects.

Answer Choice (B) So they have different ideas about medicine, I think we could have gotten that from the ‘alternative’ part.

Answer Choice (C) That’s nice for the patient and could maybe be construed as an effect (though only for the subset whom orthodox medicine couldn’t cure), but this is a much weaker answer than D.

Correct Answer Choice (D) This suggests that alternative medicine can have side effects!

Answer Choice (E) Our job is to weaken the conclusion that alternative medicine has no side effects, we don’t care about orthodox medicine’s effects!


4 comments

This page shows a recording of a live class. We're working hard to create our standard, concise explanation videos for the questions in this PrepTest. Thank you for your patience!

We should recognize this as a strengthening question, since the stem states: Which one of the following, if true, most strengthens the teacher’s argument?

Our stimulus takes the form of a dialogue between a student and teacher, and we are tasked with strengthening the teacher’s argument.

The teacher argues that journalists who don’t disclose the identity of their source stake their reputation on the ‘logic of anecdotes’. Basically, you judge their reporting, and consequently their reputation, on a similar basis to how you judge an anecdote. He supports this claim with some more specific information comparing anonymously sourced reports and anecdotes. It is necessary for an anonymous report to be published, and an anecdote to be good, that they be highly plausible, original, or interesting. The student responds by saying that if this were true, journalists wouldn’t need actual sources, since it wouldn’t be hard for a resourceful journalist to just invent plausible, original, or interesting stories. We want to support the Teacher’s claim that journalist stake their reputation on the logic of anecdotes when they use anonymous sources. On to the answer choices:

Correct Answer Choice (A) This connects our journalists premise about the requirements to be published with his conclusion about reputation.

Answer Choice (B) Our teacher’s argument is about anonymous sources; this isn’t relevant.

Answer Choice (C) This wouldn’t affect whether they get published, and hence whether the journalist stakes his reputation.

Answer Choice (D) This does nothing for us.

Answer Choice (E) We are interested in their reputation, and whether they stake it, not whether they are valued by their publishers.


Comment on this

This page shows a recording of a live class. We're working hard to create our standard, concise explanation videos for the questions in this PrepTest. Thank you for your patience!

This is a strengthening question: Which one of the following, if true, provides the most support for the argument in the passage?

The first thing we learn is that Zeria is severing its diplomatic relations with Nandalo ostensibly because of its human rights violations. However, hypocritically, Zeria continues to maintain relations with countries known to have a far worse human rights record than Nandalo. Based on this, the author concludes that this decision to sever diplomatic ties cannot be because of Zeria’s commitment to human rights. We are essentially being given an explanation for an event, that Zeria cut ties because of human rights violations, and being asked to support the conclusion that this explanation is not true. In this case I think it might be helpful to treat this like a weakening question; we want to weaken a hypothesis, so introducing an alternate hypothesis would be a great answer! Let’s see our options:

Correct Answer Choice (A) This gives Zeria another motivation to sever diplomatic ties, and consequently is an excellent alternate hypothesis. Zeria cut ties because of foreign pressure, not human rights violations.

Answer Choice (B) This does nothing to support that Zeria had other motives.

Answer Choice (C) Who cares what countries have expressed concern about.

Answer Choice (D) This doesn’t provide any support that Zeria didn’t sever diplomatic ties because of human rights violations.

Answer Choice (E) Who cares about the opposition party’s policy. Why did the government of Zeria sever diplomatic relations?


Comment on this

This page shows a recording of a live class. We're working hard to create our standard, concise explanation videos for the questions in this PrepTest. Thank you for your patience!

Here we have a strengthening questions: Which one of the following most strongly supports the explanation given in the argument?

Our stimulus begins with a phenomenon; distemper virus has caused two thirds of the seal population of the north sea to die since 1988. Our author then offers the hypothesis that the reason the normally latent virus has acted up is that pollution weakened the seals immune systems. Since this is a strengthening question and we have a hypothesis, a great answer would be one which eliminates an alternative explanation for why the virus killed so many seals, or introduces more information consistent with the hypothesis. Let’s see what we get!

Correct Answer Choice (A) If pollution is weakening the immune system of the seals, we would expect it might be doing this to other animals! This answer introduces a result we would expect if our hypothesis was true, and therefore strengthens it.

Answer Choice (B) Good for them, but this doesn’t strengthen our hypothesis. If anything it weakens it since our hypothesis requires “severe” pollution and this suggests pollution could be much better than it was before.

Answer Choice (C) I certainly hope the pollution isn’t that bad then, but this does nothing for our argument.

Answer Choice (D) This is just a factoid.

Answer Choice (E) This weakens our argument by introducing an alternate explanation.


1 comment

This page shows a recording of a live class. We're working hard to create our standard, concise explanation videos for the questions in this PrepTest. Thank you for your patience!

Here we have a most strongly supported question: The statements above, if true, most strongly support which one of the following conclusions?

The first thing the stimulus tells us is that there is a growing discrepancy between the punishments for famous and unknown defendants convicted of the same crime, where the famous person gets off with community service while the unknown person almost always gets real prison time. So your average Joe gets 5 years in prison for his DUI, but the new famous singer gets a couple months picking up trash; seems pretty unfair! The stimulus ends by reminding us that the principle of equality before the law demands that fame and publicity are put to the side when deciding a case. So the law is supposed to consider the crime itself and not whether the person who committed it is a celebrity. And that’s all we learn! We’re looking for a conclusion that would be a good fit for these premises; it will follow without us having to make any unreasonable assumptions. Let’s see what we get:

Answer Choice (A) We have reason to believe it is not being applied in some cases, but to say it is only being applied in a few cases requires a wild assumption; namely that just because this one subset of cases (cases with famous defendants) seems to be increasingly breaking the principle, that most cases are breaking it! We unfortunately just don’t know anything about cases other than these well-publicized trials; we always want to avoid concluding about things we haven’t been told anything about on MSS questions.

Answer Choice (B) Although their fame should not affect the decision concerning their punishment, that doesn’t mean there needs to be the same outcome as people without fame. Maybe celebrities tend to commit worse crimes so they should be getting more prison time even if we treat them the same as non-celebrities (i.e. judge their crimes rather than their fame).

Answer Choice (C) We have been given no information about any principles that can override it.

Correct Answer Choice (D) The growing variance in punishments for similar crimes suggests that in some cases celebrities are getting special treatment, which would break the principle of equality.

Answer Choice (E) Treating everyone equally doesn’t mean you can never be lenient depending on the case. It does mean that you can’t be lenient just because someone is famous, but we haven’t been told about anything else the principle rules out. To draw this conclusion requires we assume that the principle exclude any other possible reason for leniency; that’s a big assumption.


2 comments

This page shows a recording of a live class. We're working hard to create our standard, concise explanation videos for the questions in this PrepTest. Thank you for your patience!

This is a weakening question: Which one of the following, if true, would cast doubt on the experimenters’ conclusion?

Our stimulus begins with a definition of nuclear fusion; nuclear fusion is a process where nuclei fuse and release energy. We then learn that this process creates a by-product; helium-4 gas. Having established this context, we now learn about an experiment involving “heavy” water. I don’t know about you, but I have no idea what “heavy” water is; luckily the LSAT is about our reasoning skills and not chemistry. We learn that the water is contained in a sealed flask within an air-filled chamber, and that after the experiment some Helium-4 was found in the chamber. The people running the experiment concluded that there must have been nuclear fusion that happened. Since this is a weakening question involving a hypothesis, we should look for an alternate hypothesis in the answer choices, an alternate possible explanation for why there was helium-4 in the chamber. Let’s see what we get:

Answer Choice (A) The researcher’s explanation is entirely compatible with this.

Answer Choice (B) But was it fusion that produced the helium-4?

Correct Answer Choice (C) We are told the chamber was air-filled, so we would therefore expect that level of helium-4. This is a much better explanation than that nuclear fusion occurred!

Answer Choice (D) But it was helium-4 they found. If anything this suggests the helium-4 was recently produced which might support the fusion hypothesis.

Answer Choice (E) Cool, but we weren’t told anything about heat; for all we know there was a large release of heat and our experiment runners hypothesis is entirely correct.


Comment on this

This page shows a recording of a live class. We're working hard to create our standard, concise explanation videos for the questions in this PrepTest. Thank you for your patience!

Here we have a weakening question, indicated by: Which one of the following, if true, would tend to invalidate use of the ratings for the agency’s purpose?

So we’ve got a government rating system where the highest ranked airlines are the ones with the lowest proportion of late flights. So the number one airline would be the one whose ratio of total flights to late flights is the lowest. From the question stem, we know we want to weaken the agency’s reasoning for why this rating would be good for their purpose. We’re next told what this purpose is; the agency thinks the rating can establish an objective measure of the efficiency of the airlines personnel in meeting flight schedules. So the conclusion we want to weaken here is that the proportion of late flights actually represents how efficient the staff of each airline are. We can think of this as hypothesis; the agency believes the reason airlines have more or less late flights is the efficiency of their staff. A good answer choice will be one which introduces an alternate explanation for why airlines are late. On to the answers:

Answer Choice (A) Ok, but this wouldn’t affect whether the staff are what determine flights being late.

Correct Answer Choice (B) If flights are often made late by weather, and some airlines are more affected than others, then it is unlikely the rating would be representative of staff performance because a lot of the time it’s out of their control.

Answer Choice (C) This gives us a reason why being late isn’t always the end of the world, but has no bearing on whether the lateness of flights reflects staff performance.

Answer Choice (D) This is totally compatible with what we want to weaken.

Answer Choice (E) Who cares how we define a late flight? Our interest is in whether lateness represents inefficiency on the part of flight staff.


1 comment

This page shows a recording of a live class. We're working hard to create our standard, concise explanation videos for the questions in this PrepTest. Thank you for your patience!

This is both a strengthening and a weakening question, as we are tasked with strengthening one position and weakening another in one fell swoop: Which one of the following, if it occurred, would be the strongest evidence favoring Ms. Fring’s position over Mr. Blatt’s position?

Our stimulus is a dialogue between a Mr. Blatt and Ms. Fring. Blatt argues that expert consultants are worth their expensive fees because they help executives make better decisions. Fring is having none of this, and instead argues that consultants are hired to help executives escape responsibility, and are only high paid so that more blame can be laid on them when things go wrong. We want an answer choice that will weaken Blatt’s high demand hypothesis while supporting Fring’s fall-guy hypothesis. Let’s take a look at our options:

Answer Choice (A) There is nothing to suggest they are hiring him to take the blame if something goes wrong.

Answer Choice (B) This gives us practically no real information.

Correct Answer Choice (C) Fring’s hypothesis suggests that consultants are hired because of their expensive fees, while Blatt’s suggests it is simply a function of supply/demand. A consultant company lowering fees and losing business suggests they were being hired because of their fees, while also being a completely confusing result if the consultants were being hired for their skills; surely more people would hire them for those skills if they were cheaper! This supports Fring’s belief that it’s their fees which consultants are hired for, and is inconsistent with Blatt’s explanation.

Answer Choice (D) This suggests the consultant is being hired for his skills, not his price.

Answer Choice (E) What this answer is really missing is any information about whether the blame was laid on the consultant; but regardless marginally profitable is still profitable!


1 comment

This page shows a recording of a live class. We're working hard to create our standard, concise explanation videos for the questions in this PrepTest. Thank you for your patience!

We should recognize that this is a weakening question, as we are introducing information which weakens reasoning in the stimulus: Which one of the following, if true, is the strongest defense against the counterexample of dogs that shake hands?

Our stimulus begins with the position of brain studies; they suggest that while humans are majority right-handed, approximately half of any given group of animals will be left-handed. Our author’s conclusion is that this finding is suspect, or in other words likely wrong, because dogs will usually shake hands with their right paw. Our job is to introduce a premise which weakens the author’s criticism of the brain studies. On to the answers:

Answer Choice (A) This wouldn’t undermine whether or not they exhibit a consistent preference for a right limb in certain cases.

Answer Choice (B) The front paw is a limb!

Answer Choice (C) This might explain the results of the brain studies (specifically why animals differ from humans), but it wouldn’t weaken the author’s criticism; why do dogs shake a paw with their right paw?

Answer Choice (D) Their ability to compensate is entirely compatible with their preferring their right limb.

Correct Answer Choice (E) If humans have a preference for their right limb, when they train their dog to shake a paw they could be training them to do so with their right paw, so that although the dogs have preferences compatible with the brain study, human influence leads to more dogs shaking with their right paw.


Comment on this

This page shows a recording of a live class. We're working hard to create our standard, concise explanation videos for the questions in this PrepTest. Thank you for your patience!

This is a weakening question, as the question stem asks: Which one of the following, if true, casts the most doubt on the author’s hypothesis?

This is a fairly straight forward correlation-causation argument. A study finds that smokers are more likely to snore, and concludes that smoking can cause snoring. Our job is to weaken this argument. Let’s see what we get:

Correct Answer Choice (A) This provides an explanation for why snoring and smoking would correlate, even when smoking might have no influence on snoring, namely that the two share a third cause, stress.

Answer Choice (B) Ok, but unless we know obesity leads to snoring this does nothing for us.

Answer Choice (C) This is completely compatible with the argument.

Answer Choice (D) Same as C.

Answer Choice (E) Interesting! But this could be true while smoking caused snoring, and does nothing to suggest it doesn’t.


Comment on this